LSAT and Law School Admissions Forum

Get expert LSAT preparation and law school admissions advice from PowerScore Test Preparation.

 ellenb
  • Posts: 260
  • Joined: Oct 22, 2012
|
#10582
Dear Powerscore,

For this question I picked answer choice B and I know that the right answer is answer choice D, please let me know why D is the right answer?

Actually, I did not like none of the answers after reading them. However, I picked B since, the population of Chetahs is decreasing and their natural environment is decreasing I assumed that this might be the right answer. However, it is a must be true question and I know that I must prove it. However, as I mentioned above I did not like none of the answers.

However, I did not like D either because it is sort of looking into the future, even though it is the near future, it is still the future, and making a prognosis that we can say might be true but we do not know for sure 100%.

I hope my thought process is clear, I just want to make sure that it makes sense to me why one answer is wrong and the other one is right.

Thanks

Ellen
 David Boyle
PowerScore Staff
  • PowerScore Staff
  • Posts: 836
  • Joined: Jun 07, 2013
|
#10602
ellenb wrote:Dear Powerscore,

For this question I picked answer choice B and I know that the right answer is answer choice D, please let me know why D is the right answer?

Actually, I did not like none of the answers after reading them. However, I picked B since, the population of Chetahs is decreasing and their natural environment is decreasing I assumed that this might be the right answer. However, it is a must be true question and I know that I must prove it. However, as I mentioned above I did not like none of the answers.

However, I did not like D either because it is sort of looking into the future, even though it is the near future, it is still the future, and making a prognosis that we can say might be true but we do not know for sure 100%.

I hope my thought process is clear, I just want to make sure that it makes sense to me why one answer is wrong and the other one is right.

Thanks

Ellen
Hello Ellen,

As for B, "The cheetah’s natural habitat is decreasing in size at a faster rate than is the size of the wild cheetah population", we have no way of knowing that.
D is right because the stimulus basically says, "snd arrow mc" (surviving natural disaster needs more cheetahs than there are right now), and, because of "At present . . . there is not enough African grassland to support a wild cheetah population larger than the current population", "slash mc" (there aren't going to be more cheetahs for a while). From the contrapositive, then we get "slash mc arrow slash snd", and "slash snd" is answer D, basically ("In the short term, the wild cheetah population will be incapable of surviving a natural disaster in the African grasslands.").
By the way, a prognosis is not a problem if it's proven by the stimulus, as answer D is proven by the stimulus.
Hope that helps,

David
 ellenb
  • Posts: 260
  • Joined: Oct 22, 2012
|
#10641
Thanks David,

How did you know that you need to put it into a sufficient and necessary condition?, was it because of "in order for" It did not occur to me to put into a sufficient and necessary condition. I suppose I did not see any word indicators to do that.

Thanks

Ellen
 David Boyle
PowerScore Staff
  • PowerScore Staff
  • Posts: 836
  • Joined: Jun 07, 2013
|
#10664
ellenb wrote:Thanks David,

How did you know that you need to put it into a sufficient and necessary condition?, was it because of "in order for" It did not occur to me to put into a sufficient and necessary condition. I suppose I did not see any word indicators to do that.

Thanks

Ellen
Hello,

"In order to" or "In order for" is a "sufficient" indicator. It's another way of saying "if". ("If the cheetahs are to survive a natural disaster, there must be more cheetahs.")

David
 srcline@noctrl.edu
  • Posts: 243
  • Joined: Oct 16, 2015
|
#23050
Hello

So this is more of a clarification question. Am I correct in saying that both the questions stem for 17 and 18 are Strengthen questions.

17: The statements above, if true most strongly support which one of the following conclusions?

18: The principle above, if valid most strongly supports which on of the following arguments.

So the stimulus contains the Suf/ Nec logic and I diagrammed this as follows

IF a work of art, truly great :arrow: Org. and FARINF on ARTCOMM.

Contra (+) (not) orginal or FARINF :arrow: not truly great

SO would B, be correct b/c its the contrapostive of this statement?

I am having some major confusion when it comes to the question stems of MBT and MSS and Strengthen questions.
Thankyou
Sarah
 Robert Carroll
PowerScore Staff
  • PowerScore Staff
  • Posts: 1787
  • Joined: Dec 06, 2013
|
#23070
Sarah,

These are both Must Be True questions. Remember that MBT questions are in the First Family, and that the information flows downward. The questions say that the stimulus is to be trusted - "The statements above, if true..." and "The principle above, if valid..." You are to find an answer that follows from that stimulus. The stimulus information supports the answer choice.

In a Strengthen question, the answer choice supports the argument of the stimulus. That is why new information is fine in a Strengthen question answer choice, but unacceptable in a Must Be True question answer choice.

I think your contrapositive is ok, but the "not" is a bit ambiguous in the contrapositive, so let me clarify (and if you meant what I'm about to say, that's great!). The sufficient condition of the contrapositive should be "not original or not FARINF". Each thing is negated.

Answer choice (B) is indeed correct via the contrapositive of the stimulus.

Robert Carroll

Get the most out of your LSAT Prep Plus subscription.

Analyze and track your performance with our Testing and Analytics Package.